www.vorkurse.de
Ein Projekt von vorhilfe.de
Die Online-Kurse der Vorhilfe

E-Learning leicht gemacht.
Hallo Gast!einloggen | registrieren ]
Startseite · Mitglieder · Teams · Forum · Wissen · Kurse · Impressum
Forenbaum
^ Forenbaum
Status Mathe-Vorkurse
  Status Organisatorisches
  Status Schule
    Status Wiederholung Algebra
    Status Einführung Analysis
    Status Einführung Analytisc
    Status VK 21: Mathematik 6.
    Status VK 37: Kurvendiskussionen
    Status VK Abivorbereitungen
  Status Universität
    Status Lerngruppe LinAlg
    Status VK 13 Analysis I FH
    Status Algebra 2006
    Status VK 22: Algebra 2007
    Status GruMiHH 06
    Status VK 58: Algebra 1
    Status VK 59: Lineare Algebra
    Status VK 60: Analysis
    Status Wahrscheinlichkeitst

Gezeigt werden alle Foren bis zur Tiefe 2

Navigation
 Startseite...
 Neuerdings beta neu
 Forum...
 vorwissen...
 vorkurse...
 Werkzeuge...
 Nachhilfevermittlung beta...
 Online-Spiele beta
 Suchen
 Verein...
 Impressum
Das Projekt
Server und Internetanbindung werden durch Spenden finanziert.
Organisiert wird das Projekt von unserem Koordinatorenteam.
Hunderte Mitglieder helfen ehrenamtlich in unseren moderierten Foren.
Anbieter der Seite ist der gemeinnützige Verein "Vorhilfe.de e.V.".
Partnerseiten
Weitere Fächer:

Open Source FunktionenplotterFunkyPlot: Kostenloser und quelloffener Funktionenplotter für Linux und andere Betriebssysteme
Forum "Uni-Komplexe Analysis" - Konvergenzradius
Konvergenzradius < komplex < Analysis < Hochschule < Mathe < Vorhilfe
Ansicht: [ geschachtelt ] | ^ Forum "Uni-Komplexe Analysis"  | ^^ Alle Foren  | ^ Forenbaum  | Materialien

Konvergenzradius: Frage (beantwortet)
Status: (Frage) beantwortet Status 
Datum: 10:11 Fr 08.05.2015
Autor: rollroll

Aufgabe
Berechne den Konvergenzradius von [mm] \summe_{n=1}^{\infty}z^{n!}. [/mm]


Hallo,

ich habe folgendes gemacht:
Definiere für k [mm] \in [/mm] IN
[mm] a_k= [/mm] 1, falls ein n [mm] \in [/mm] IN existiert mit k=n! und [mm] a_k=0, [/mm] sonst

Dann ist [mm] \summe_{n=1}^{\infty}z^{n!} [/mm] = [mm] \summe_{k=1}^{\infty}a_kz^{k}. [/mm] Wegen [mm] \limes_{k\rightarrow\infty} \wurzel[k]{|a_k|}=1 [/mm] ist der Konvergenzradius 1.

Was meint ihr?


        
Bezug
Konvergenzradius: Antwort
Status: (Antwort) fertig Status 
Datum: 10:19 Fr 08.05.2015
Autor: fred97


> Berechne den Konvergenzradius von
> [mm]\summe_{n=1}^{\infty}z^{n!}.[/mm]
>  Hallo,
>  
> ich habe folgendes gemacht:
>  Definiere für k [mm]\in[/mm] IN
>  [mm]a_k=[/mm] 1, falls ein n [mm]\in[/mm] IN existiert mit k=n! und [mm]a_k=0,[/mm]
> sonst
>  
> Dann ist [mm]\summe_{n=1}^{\infty}z^{n!}[/mm] =
> [mm]\summe_{k=1}^{\infty}a_kz^{k}.[/mm]

Das ist O.K.



>  Wegen
> [mm]\limes_{k\rightarrow\infty} \wurzel[k]{|a_k|}=1[/mm]


Das ist nicht O.K.

Die Folge [mm] (\wurzel[k]{|a_k|})_{k \in \IN} [/mm] ist nicht konvergent ! Aber es ist

   [mm]lim sup_{k\rightarrow\infty} \wurzel[k]{|a_k|}=1[/mm]



> ist der
> Konvergenzradius 1.

Ja

FRED

>  
> Was meint ihr?
>  


Bezug
        
Bezug
Konvergenzradius: Antwort
Status: (Antwort) fertig Status 
Datum: 14:02 Fr 08.05.2015
Autor: Marcel

Hallo,

> Berechne den Konvergenzradius von
> [mm]\summe_{n=1}^{\infty}z^{n!}.[/mm]

Du hast also

    [mm] $\summe_{n=1}^{\infty}\blue{1}z^{n!}$ [/mm]

Du kannst auch (mache Dir das klar) den Konvergenzradius per

    [mm] $\limsup_{n \to \infty}\sqrt[\red{n!}]{\blue{1}}=\limsup_{n \to \infty}\blue{1}^{1/\red{n!}}$ [/mm]

berechnen.

Siehe auch

    https://matheraum.de/read?i=1048546

Gruß,
  Marcel

Bezug
        
Bezug
Konvergenzradius: Mitteilung
Status: (Mitteilung) Reaktion unnötig Status 
Datum: 14:28 Fr 08.05.2015
Autor: Thomas_Aut

Hallo,

Sollte dir jemals langweilig sein:

Zu dieser 'Lückenreihe' kann man (weil du das auch ins komplexe Analyis Forum schreibst) einige interessante Aufgaben stellen - unter anderem , kann man sich überlegen, dass sie am Rand nicht analytisch fortgesetzt werden kann:

also es gibt kein Gebiet $D [mm] \supset \mathbb{D}$ [/mm] wobei [mm] $\mathbb{D}:=\{z \in \mathbb{C} : |z| < 1\}$sodass [/mm] f eine analytische Fortsetzung g auf D hat.


Das ist gar nicht so einfach.


LG

Bezug
        
Bezug
Konvergenzradius: Antwort
Status: (Antwort) fertig Status 
Datum: 18:16 Fr 08.05.2015
Autor: fred97

Man kann es auch so machen:

für |z|<1 ist [mm] |z|^{n!} \le |z|^n, [/mm] also konvergiert Pdie otenzreihe für diese z.

Für |z| [mm] \ge [/mm] 1 ist [mm] (|z|^{n!}) [/mm] kein Nullfolge

FRED

Bezug
Ansicht: [ geschachtelt ] | ^ Forum "Uni-Komplexe Analysis"  | ^^ Alle Foren  | ^ Forenbaum  | Materialien


^ Seitenanfang ^
www.vorkurse.de
[ Startseite | Mitglieder | Teams | Forum | Wissen | Kurse | Impressum ]